Factors in a different base $ 2b^2!+!9b!+!7,mid, 7b^2!+!9b!+!2$












6












$begingroup$



Two numbers $297_B$ and $792_B$, belong to base $B$ number system. If the first number is a factor of the second number, then what is the value of $B$?




Solution:
enter image description here



But base cannot be negative. Could someone please explain where I am going wrong?










share|cite|improve this question











$endgroup$

















    6












    $begingroup$



    Two numbers $297_B$ and $792_B$, belong to base $B$ number system. If the first number is a factor of the second number, then what is the value of $B$?




    Solution:
    enter image description here



    But base cannot be negative. Could someone please explain where I am going wrong?










    share|cite|improve this question











    $endgroup$















      6












      6








      6


      2



      $begingroup$



      Two numbers $297_B$ and $792_B$, belong to base $B$ number system. If the first number is a factor of the second number, then what is the value of $B$?




      Solution:
      enter image description here



      But base cannot be negative. Could someone please explain where I am going wrong?










      share|cite|improve this question











      $endgroup$





      Two numbers $297_B$ and $792_B$, belong to base $B$ number system. If the first number is a factor of the second number, then what is the value of $B$?




      Solution:
      enter image description here



      But base cannot be negative. Could someone please explain where I am going wrong?







      elementary-number-theory divisibility number-systems






      share|cite|improve this question















      share|cite|improve this question













      share|cite|improve this question




      share|cite|improve this question








      edited Dec 31 '18 at 21:51









      Maria Mazur

      50.3k1361126




      50.3k1361126










      asked Dec 31 '18 at 20:29









      Aamir KhanAamir Khan

      455




      455






















          4 Answers
          4






          active

          oldest

          votes


















          10












          $begingroup$

          The long division is the source of the error; you can't have $7/2$ as the quotient. The quotient needs to be an integer, that's what "factor" means.



          If the quotient is $2$, then the base is $4$. This is found by solving $7B^2+9B+2=color{red}{ 2}(2B^2+9B+7)$, and discarding the negative root.



          If the quotient is $3$, then the base is $19$. This is found by solving $7B^2+9B+2=color{red}{ 3}(2B^2+9B+7)$, and discarding the negative root.



          No other quotients make any sense. However, if the base is $4$, then you don't get digits $7$ and $9$. Hence the answer must be $B=19$.






          share|cite|improve this answer









          $endgroup$













          • $begingroup$
            Thank you so much. This was really very helpful. :)
            $endgroup$
            – Aamir Khan
            Dec 31 '18 at 20:37










          • $begingroup$
            My pleasure, glad to help.
            $endgroup$
            – vadim123
            Dec 31 '18 at 20:39










          • $begingroup$
            @AamirKhan Beware that the above is not a rigorous solution without proof that those are the only possible quotients. See my answer for one rigorous approach
            $endgroup$
            – Bill Dubuque
            Dec 31 '18 at 21:26








          • 1




            $begingroup$
            @BillDubuque, it's not difficult to make it rigorous, to prove that the quotient $q$ can't be bigger than $3$. $(2q-7)B^2+(9q-9)B+(7q-2)$ is strictly positive if $qge 4$ and $B>0$. Hence if $qge 4$, no positive $B$ solves the equation. Similarly, if $q=1$, then the only positive solution is $B=1$, which is not possible.
            $endgroup$
            – vadim123
            Dec 31 '18 at 21:48












          • $begingroup$
            @vadim You should add the details of a rigorous proof to the answer (I've lost count of the number of times "[this case] doesn't make sense" turned out to be incorrect), so we should not encourage students to handwave like that.
            $endgroup$
            – Bill Dubuque
            Dec 31 '18 at 21:58





















          6












          $begingroup$

          Going $1$ step more with Euclid's algorithm reveals a common factor $,b!+!1.,$ Cancelling it



          $$dfrac{7b^2!+!9b!+!2}{2b^2!+!9b!+!7} = color{#c00}{dfrac{7b!+!2}{2b!+!7}}inBbb Z , Rightarrow, 7-2 dfrac{color{#c00}{7b!+!2}}{ color{#c00}{2b!+!7}}, =, dfrac{45}{2b!+!7}inBbb Zqquad$$



          Therefore $,2b!+!7mid 45 $ so $,b> 9,$(= digit) $,Rightarrow,2b!+!7 = 45,$ $Rightarrow,b=19.$






          share|cite|improve this answer











          $endgroup$





















            1












            $begingroup$

            Since $b+1>0$ and $$(b+1)(2b+7)mid (7b+2)(b+1)implies 2b+7mid 7b+2$$



            we have $$2b+7mid (7b+2)-3(2b+7) = b-19$$



            so if $b-19> 0$ we have $$2b+7mid b-19 implies 2b+7leq b-19 implies b+26leq 0$$



            which is not true. So $bleq 19$. By trial and error we see that $b=4$ and $b=19$ works.






            share|cite|improve this answer











            $endgroup$





















              0












              $begingroup$

              $$2B^2+9B+7mid 7B^2+9B+2$$



              Let's write $aB^2+bB + c$ as $[a,b,c]_B$ to emphasis that $a,b,c$ are digits base $B$.



              Then $[2,9,7]_B mid [7,9,2]_B-[2,9,7]_B$ and we are assuming that $2,9,7 < B$



              Writing this out "subtraction-style", we get



              $left.begin{array}{c}
              & 7 & 9 & 2 \
              -& 2 & 9 & 7 \
              hline
              phantom{4}
              end{array}
              right.
              implies
              left.begin{array}{c}
              & 6 & (B+8) & (B+2) \
              -& 2 & 9 & 7 \
              hline
              & 4 & (B-1) & (B-5)
              end{array}
              right.
              $



              So $[4,B-1,B-5]_B$ is a multiple of $[2,9,7]_B$.



              We must therefore have $[4,B-1,B-5]_B = 2[2,9,7]_B = [4,18,14]_B$ which implies $B-1=18$ and $B-5=14$. Hence $B=19$.






              share|cite|improve this answer











              $endgroup$














                Your Answer








                StackExchange.ready(function() {
                var channelOptions = {
                tags: "".split(" "),
                id: "69"
                };
                initTagRenderer("".split(" "), "".split(" "), channelOptions);

                StackExchange.using("externalEditor", function() {
                // Have to fire editor after snippets, if snippets enabled
                if (StackExchange.settings.snippets.snippetsEnabled) {
                StackExchange.using("snippets", function() {
                createEditor();
                });
                }
                else {
                createEditor();
                }
                });

                function createEditor() {
                StackExchange.prepareEditor({
                heartbeatType: 'answer',
                autoActivateHeartbeat: false,
                convertImagesToLinks: true,
                noModals: true,
                showLowRepImageUploadWarning: true,
                reputationToPostImages: 10,
                bindNavPrevention: true,
                postfix: "",
                imageUploader: {
                brandingHtml: "Powered by u003ca class="icon-imgur-white" href="https://imgur.com/"u003eu003c/au003e",
                contentPolicyHtml: "User contributions licensed under u003ca href="https://creativecommons.org/licenses/by-sa/3.0/"u003ecc by-sa 3.0 with attribution requiredu003c/au003e u003ca href="https://stackoverflow.com/legal/content-policy"u003e(content policy)u003c/au003e",
                allowUrls: true
                },
                noCode: true, onDemand: true,
                discardSelector: ".discard-answer"
                ,immediatelyShowMarkdownHelp:true
                });


                }
                });














                draft saved

                draft discarded


















                StackExchange.ready(
                function () {
                StackExchange.openid.initPostLogin('.new-post-login', 'https%3a%2f%2fmath.stackexchange.com%2fquestions%2f3058019%2ffactors-in-a-different-base-2b2-9b-7-mid-7b2-9b-2%23new-answer', 'question_page');
                }
                );

                Post as a guest















                Required, but never shown

























                4 Answers
                4






                active

                oldest

                votes








                4 Answers
                4






                active

                oldest

                votes









                active

                oldest

                votes






                active

                oldest

                votes









                10












                $begingroup$

                The long division is the source of the error; you can't have $7/2$ as the quotient. The quotient needs to be an integer, that's what "factor" means.



                If the quotient is $2$, then the base is $4$. This is found by solving $7B^2+9B+2=color{red}{ 2}(2B^2+9B+7)$, and discarding the negative root.



                If the quotient is $3$, then the base is $19$. This is found by solving $7B^2+9B+2=color{red}{ 3}(2B^2+9B+7)$, and discarding the negative root.



                No other quotients make any sense. However, if the base is $4$, then you don't get digits $7$ and $9$. Hence the answer must be $B=19$.






                share|cite|improve this answer









                $endgroup$













                • $begingroup$
                  Thank you so much. This was really very helpful. :)
                  $endgroup$
                  – Aamir Khan
                  Dec 31 '18 at 20:37










                • $begingroup$
                  My pleasure, glad to help.
                  $endgroup$
                  – vadim123
                  Dec 31 '18 at 20:39










                • $begingroup$
                  @AamirKhan Beware that the above is not a rigorous solution without proof that those are the only possible quotients. See my answer for one rigorous approach
                  $endgroup$
                  – Bill Dubuque
                  Dec 31 '18 at 21:26








                • 1




                  $begingroup$
                  @BillDubuque, it's not difficult to make it rigorous, to prove that the quotient $q$ can't be bigger than $3$. $(2q-7)B^2+(9q-9)B+(7q-2)$ is strictly positive if $qge 4$ and $B>0$. Hence if $qge 4$, no positive $B$ solves the equation. Similarly, if $q=1$, then the only positive solution is $B=1$, which is not possible.
                  $endgroup$
                  – vadim123
                  Dec 31 '18 at 21:48












                • $begingroup$
                  @vadim You should add the details of a rigorous proof to the answer (I've lost count of the number of times "[this case] doesn't make sense" turned out to be incorrect), so we should not encourage students to handwave like that.
                  $endgroup$
                  – Bill Dubuque
                  Dec 31 '18 at 21:58


















                10












                $begingroup$

                The long division is the source of the error; you can't have $7/2$ as the quotient. The quotient needs to be an integer, that's what "factor" means.



                If the quotient is $2$, then the base is $4$. This is found by solving $7B^2+9B+2=color{red}{ 2}(2B^2+9B+7)$, and discarding the negative root.



                If the quotient is $3$, then the base is $19$. This is found by solving $7B^2+9B+2=color{red}{ 3}(2B^2+9B+7)$, and discarding the negative root.



                No other quotients make any sense. However, if the base is $4$, then you don't get digits $7$ and $9$. Hence the answer must be $B=19$.






                share|cite|improve this answer









                $endgroup$













                • $begingroup$
                  Thank you so much. This was really very helpful. :)
                  $endgroup$
                  – Aamir Khan
                  Dec 31 '18 at 20:37










                • $begingroup$
                  My pleasure, glad to help.
                  $endgroup$
                  – vadim123
                  Dec 31 '18 at 20:39










                • $begingroup$
                  @AamirKhan Beware that the above is not a rigorous solution without proof that those are the only possible quotients. See my answer for one rigorous approach
                  $endgroup$
                  – Bill Dubuque
                  Dec 31 '18 at 21:26








                • 1




                  $begingroup$
                  @BillDubuque, it's not difficult to make it rigorous, to prove that the quotient $q$ can't be bigger than $3$. $(2q-7)B^2+(9q-9)B+(7q-2)$ is strictly positive if $qge 4$ and $B>0$. Hence if $qge 4$, no positive $B$ solves the equation. Similarly, if $q=1$, then the only positive solution is $B=1$, which is not possible.
                  $endgroup$
                  – vadim123
                  Dec 31 '18 at 21:48












                • $begingroup$
                  @vadim You should add the details of a rigorous proof to the answer (I've lost count of the number of times "[this case] doesn't make sense" turned out to be incorrect), so we should not encourage students to handwave like that.
                  $endgroup$
                  – Bill Dubuque
                  Dec 31 '18 at 21:58
















                10












                10








                10





                $begingroup$

                The long division is the source of the error; you can't have $7/2$ as the quotient. The quotient needs to be an integer, that's what "factor" means.



                If the quotient is $2$, then the base is $4$. This is found by solving $7B^2+9B+2=color{red}{ 2}(2B^2+9B+7)$, and discarding the negative root.



                If the quotient is $3$, then the base is $19$. This is found by solving $7B^2+9B+2=color{red}{ 3}(2B^2+9B+7)$, and discarding the negative root.



                No other quotients make any sense. However, if the base is $4$, then you don't get digits $7$ and $9$. Hence the answer must be $B=19$.






                share|cite|improve this answer









                $endgroup$



                The long division is the source of the error; you can't have $7/2$ as the quotient. The quotient needs to be an integer, that's what "factor" means.



                If the quotient is $2$, then the base is $4$. This is found by solving $7B^2+9B+2=color{red}{ 2}(2B^2+9B+7)$, and discarding the negative root.



                If the quotient is $3$, then the base is $19$. This is found by solving $7B^2+9B+2=color{red}{ 3}(2B^2+9B+7)$, and discarding the negative root.



                No other quotients make any sense. However, if the base is $4$, then you don't get digits $7$ and $9$. Hence the answer must be $B=19$.







                share|cite|improve this answer












                share|cite|improve this answer



                share|cite|improve this answer










                answered Dec 31 '18 at 20:35









                vadim123vadim123

                76.7k899191




                76.7k899191












                • $begingroup$
                  Thank you so much. This was really very helpful. :)
                  $endgroup$
                  – Aamir Khan
                  Dec 31 '18 at 20:37










                • $begingroup$
                  My pleasure, glad to help.
                  $endgroup$
                  – vadim123
                  Dec 31 '18 at 20:39










                • $begingroup$
                  @AamirKhan Beware that the above is not a rigorous solution without proof that those are the only possible quotients. See my answer for one rigorous approach
                  $endgroup$
                  – Bill Dubuque
                  Dec 31 '18 at 21:26








                • 1




                  $begingroup$
                  @BillDubuque, it's not difficult to make it rigorous, to prove that the quotient $q$ can't be bigger than $3$. $(2q-7)B^2+(9q-9)B+(7q-2)$ is strictly positive if $qge 4$ and $B>0$. Hence if $qge 4$, no positive $B$ solves the equation. Similarly, if $q=1$, then the only positive solution is $B=1$, which is not possible.
                  $endgroup$
                  – vadim123
                  Dec 31 '18 at 21:48












                • $begingroup$
                  @vadim You should add the details of a rigorous proof to the answer (I've lost count of the number of times "[this case] doesn't make sense" turned out to be incorrect), so we should not encourage students to handwave like that.
                  $endgroup$
                  – Bill Dubuque
                  Dec 31 '18 at 21:58




















                • $begingroup$
                  Thank you so much. This was really very helpful. :)
                  $endgroup$
                  – Aamir Khan
                  Dec 31 '18 at 20:37










                • $begingroup$
                  My pleasure, glad to help.
                  $endgroup$
                  – vadim123
                  Dec 31 '18 at 20:39










                • $begingroup$
                  @AamirKhan Beware that the above is not a rigorous solution without proof that those are the only possible quotients. See my answer for one rigorous approach
                  $endgroup$
                  – Bill Dubuque
                  Dec 31 '18 at 21:26








                • 1




                  $begingroup$
                  @BillDubuque, it's not difficult to make it rigorous, to prove that the quotient $q$ can't be bigger than $3$. $(2q-7)B^2+(9q-9)B+(7q-2)$ is strictly positive if $qge 4$ and $B>0$. Hence if $qge 4$, no positive $B$ solves the equation. Similarly, if $q=1$, then the only positive solution is $B=1$, which is not possible.
                  $endgroup$
                  – vadim123
                  Dec 31 '18 at 21:48












                • $begingroup$
                  @vadim You should add the details of a rigorous proof to the answer (I've lost count of the number of times "[this case] doesn't make sense" turned out to be incorrect), so we should not encourage students to handwave like that.
                  $endgroup$
                  – Bill Dubuque
                  Dec 31 '18 at 21:58


















                $begingroup$
                Thank you so much. This was really very helpful. :)
                $endgroup$
                – Aamir Khan
                Dec 31 '18 at 20:37




                $begingroup$
                Thank you so much. This was really very helpful. :)
                $endgroup$
                – Aamir Khan
                Dec 31 '18 at 20:37












                $begingroup$
                My pleasure, glad to help.
                $endgroup$
                – vadim123
                Dec 31 '18 at 20:39




                $begingroup$
                My pleasure, glad to help.
                $endgroup$
                – vadim123
                Dec 31 '18 at 20:39












                $begingroup$
                @AamirKhan Beware that the above is not a rigorous solution without proof that those are the only possible quotients. See my answer for one rigorous approach
                $endgroup$
                – Bill Dubuque
                Dec 31 '18 at 21:26






                $begingroup$
                @AamirKhan Beware that the above is not a rigorous solution without proof that those are the only possible quotients. See my answer for one rigorous approach
                $endgroup$
                – Bill Dubuque
                Dec 31 '18 at 21:26






                1




                1




                $begingroup$
                @BillDubuque, it's not difficult to make it rigorous, to prove that the quotient $q$ can't be bigger than $3$. $(2q-7)B^2+(9q-9)B+(7q-2)$ is strictly positive if $qge 4$ and $B>0$. Hence if $qge 4$, no positive $B$ solves the equation. Similarly, if $q=1$, then the only positive solution is $B=1$, which is not possible.
                $endgroup$
                – vadim123
                Dec 31 '18 at 21:48






                $begingroup$
                @BillDubuque, it's not difficult to make it rigorous, to prove that the quotient $q$ can't be bigger than $3$. $(2q-7)B^2+(9q-9)B+(7q-2)$ is strictly positive if $qge 4$ and $B>0$. Hence if $qge 4$, no positive $B$ solves the equation. Similarly, if $q=1$, then the only positive solution is $B=1$, which is not possible.
                $endgroup$
                – vadim123
                Dec 31 '18 at 21:48














                $begingroup$
                @vadim You should add the details of a rigorous proof to the answer (I've lost count of the number of times "[this case] doesn't make sense" turned out to be incorrect), so we should not encourage students to handwave like that.
                $endgroup$
                – Bill Dubuque
                Dec 31 '18 at 21:58






                $begingroup$
                @vadim You should add the details of a rigorous proof to the answer (I've lost count of the number of times "[this case] doesn't make sense" turned out to be incorrect), so we should not encourage students to handwave like that.
                $endgroup$
                – Bill Dubuque
                Dec 31 '18 at 21:58













                6












                $begingroup$

                Going $1$ step more with Euclid's algorithm reveals a common factor $,b!+!1.,$ Cancelling it



                $$dfrac{7b^2!+!9b!+!2}{2b^2!+!9b!+!7} = color{#c00}{dfrac{7b!+!2}{2b!+!7}}inBbb Z , Rightarrow, 7-2 dfrac{color{#c00}{7b!+!2}}{ color{#c00}{2b!+!7}}, =, dfrac{45}{2b!+!7}inBbb Zqquad$$



                Therefore $,2b!+!7mid 45 $ so $,b> 9,$(= digit) $,Rightarrow,2b!+!7 = 45,$ $Rightarrow,b=19.$






                share|cite|improve this answer











                $endgroup$


















                  6












                  $begingroup$

                  Going $1$ step more with Euclid's algorithm reveals a common factor $,b!+!1.,$ Cancelling it



                  $$dfrac{7b^2!+!9b!+!2}{2b^2!+!9b!+!7} = color{#c00}{dfrac{7b!+!2}{2b!+!7}}inBbb Z , Rightarrow, 7-2 dfrac{color{#c00}{7b!+!2}}{ color{#c00}{2b!+!7}}, =, dfrac{45}{2b!+!7}inBbb Zqquad$$



                  Therefore $,2b!+!7mid 45 $ so $,b> 9,$(= digit) $,Rightarrow,2b!+!7 = 45,$ $Rightarrow,b=19.$






                  share|cite|improve this answer











                  $endgroup$
















                    6












                    6








                    6





                    $begingroup$

                    Going $1$ step more with Euclid's algorithm reveals a common factor $,b!+!1.,$ Cancelling it



                    $$dfrac{7b^2!+!9b!+!2}{2b^2!+!9b!+!7} = color{#c00}{dfrac{7b!+!2}{2b!+!7}}inBbb Z , Rightarrow, 7-2 dfrac{color{#c00}{7b!+!2}}{ color{#c00}{2b!+!7}}, =, dfrac{45}{2b!+!7}inBbb Zqquad$$



                    Therefore $,2b!+!7mid 45 $ so $,b> 9,$(= digit) $,Rightarrow,2b!+!7 = 45,$ $Rightarrow,b=19.$






                    share|cite|improve this answer











                    $endgroup$



                    Going $1$ step more with Euclid's algorithm reveals a common factor $,b!+!1.,$ Cancelling it



                    $$dfrac{7b^2!+!9b!+!2}{2b^2!+!9b!+!7} = color{#c00}{dfrac{7b!+!2}{2b!+!7}}inBbb Z , Rightarrow, 7-2 dfrac{color{#c00}{7b!+!2}}{ color{#c00}{2b!+!7}}, =, dfrac{45}{2b!+!7}inBbb Zqquad$$



                    Therefore $,2b!+!7mid 45 $ so $,b> 9,$(= digit) $,Rightarrow,2b!+!7 = 45,$ $Rightarrow,b=19.$







                    share|cite|improve this answer














                    share|cite|improve this answer



                    share|cite|improve this answer








                    edited Jan 1 at 3:46

























                    answered Dec 31 '18 at 21:12









                    Bill DubuqueBill Dubuque

                    214k29197660




                    214k29197660























                        1












                        $begingroup$

                        Since $b+1>0$ and $$(b+1)(2b+7)mid (7b+2)(b+1)implies 2b+7mid 7b+2$$



                        we have $$2b+7mid (7b+2)-3(2b+7) = b-19$$



                        so if $b-19> 0$ we have $$2b+7mid b-19 implies 2b+7leq b-19 implies b+26leq 0$$



                        which is not true. So $bleq 19$. By trial and error we see that $b=4$ and $b=19$ works.






                        share|cite|improve this answer











                        $endgroup$


















                          1












                          $begingroup$

                          Since $b+1>0$ and $$(b+1)(2b+7)mid (7b+2)(b+1)implies 2b+7mid 7b+2$$



                          we have $$2b+7mid (7b+2)-3(2b+7) = b-19$$



                          so if $b-19> 0$ we have $$2b+7mid b-19 implies 2b+7leq b-19 implies b+26leq 0$$



                          which is not true. So $bleq 19$. By trial and error we see that $b=4$ and $b=19$ works.






                          share|cite|improve this answer











                          $endgroup$
















                            1












                            1








                            1





                            $begingroup$

                            Since $b+1>0$ and $$(b+1)(2b+7)mid (7b+2)(b+1)implies 2b+7mid 7b+2$$



                            we have $$2b+7mid (7b+2)-3(2b+7) = b-19$$



                            so if $b-19> 0$ we have $$2b+7mid b-19 implies 2b+7leq b-19 implies b+26leq 0$$



                            which is not true. So $bleq 19$. By trial and error we see that $b=4$ and $b=19$ works.






                            share|cite|improve this answer











                            $endgroup$



                            Since $b+1>0$ and $$(b+1)(2b+7)mid (7b+2)(b+1)implies 2b+7mid 7b+2$$



                            we have $$2b+7mid (7b+2)-3(2b+7) = b-19$$



                            so if $b-19> 0$ we have $$2b+7mid b-19 implies 2b+7leq b-19 implies b+26leq 0$$



                            which is not true. So $bleq 19$. By trial and error we see that $b=4$ and $b=19$ works.







                            share|cite|improve this answer














                            share|cite|improve this answer



                            share|cite|improve this answer








                            edited Dec 31 '18 at 21:49

























                            answered Dec 31 '18 at 21:43









                            Maria MazurMaria Mazur

                            50.3k1361126




                            50.3k1361126























                                0












                                $begingroup$

                                $$2B^2+9B+7mid 7B^2+9B+2$$



                                Let's write $aB^2+bB + c$ as $[a,b,c]_B$ to emphasis that $a,b,c$ are digits base $B$.



                                Then $[2,9,7]_B mid [7,9,2]_B-[2,9,7]_B$ and we are assuming that $2,9,7 < B$



                                Writing this out "subtraction-style", we get



                                $left.begin{array}{c}
                                & 7 & 9 & 2 \
                                -& 2 & 9 & 7 \
                                hline
                                phantom{4}
                                end{array}
                                right.
                                implies
                                left.begin{array}{c}
                                & 6 & (B+8) & (B+2) \
                                -& 2 & 9 & 7 \
                                hline
                                & 4 & (B-1) & (B-5)
                                end{array}
                                right.
                                $



                                So $[4,B-1,B-5]_B$ is a multiple of $[2,9,7]_B$.



                                We must therefore have $[4,B-1,B-5]_B = 2[2,9,7]_B = [4,18,14]_B$ which implies $B-1=18$ and $B-5=14$. Hence $B=19$.






                                share|cite|improve this answer











                                $endgroup$


















                                  0












                                  $begingroup$

                                  $$2B^2+9B+7mid 7B^2+9B+2$$



                                  Let's write $aB^2+bB + c$ as $[a,b,c]_B$ to emphasis that $a,b,c$ are digits base $B$.



                                  Then $[2,9,7]_B mid [7,9,2]_B-[2,9,7]_B$ and we are assuming that $2,9,7 < B$



                                  Writing this out "subtraction-style", we get



                                  $left.begin{array}{c}
                                  & 7 & 9 & 2 \
                                  -& 2 & 9 & 7 \
                                  hline
                                  phantom{4}
                                  end{array}
                                  right.
                                  implies
                                  left.begin{array}{c}
                                  & 6 & (B+8) & (B+2) \
                                  -& 2 & 9 & 7 \
                                  hline
                                  & 4 & (B-1) & (B-5)
                                  end{array}
                                  right.
                                  $



                                  So $[4,B-1,B-5]_B$ is a multiple of $[2,9,7]_B$.



                                  We must therefore have $[4,B-1,B-5]_B = 2[2,9,7]_B = [4,18,14]_B$ which implies $B-1=18$ and $B-5=14$. Hence $B=19$.






                                  share|cite|improve this answer











                                  $endgroup$
















                                    0












                                    0








                                    0





                                    $begingroup$

                                    $$2B^2+9B+7mid 7B^2+9B+2$$



                                    Let's write $aB^2+bB + c$ as $[a,b,c]_B$ to emphasis that $a,b,c$ are digits base $B$.



                                    Then $[2,9,7]_B mid [7,9,2]_B-[2,9,7]_B$ and we are assuming that $2,9,7 < B$



                                    Writing this out "subtraction-style", we get



                                    $left.begin{array}{c}
                                    & 7 & 9 & 2 \
                                    -& 2 & 9 & 7 \
                                    hline
                                    phantom{4}
                                    end{array}
                                    right.
                                    implies
                                    left.begin{array}{c}
                                    & 6 & (B+8) & (B+2) \
                                    -& 2 & 9 & 7 \
                                    hline
                                    & 4 & (B-1) & (B-5)
                                    end{array}
                                    right.
                                    $



                                    So $[4,B-1,B-5]_B$ is a multiple of $[2,9,7]_B$.



                                    We must therefore have $[4,B-1,B-5]_B = 2[2,9,7]_B = [4,18,14]_B$ which implies $B-1=18$ and $B-5=14$. Hence $B=19$.






                                    share|cite|improve this answer











                                    $endgroup$



                                    $$2B^2+9B+7mid 7B^2+9B+2$$



                                    Let's write $aB^2+bB + c$ as $[a,b,c]_B$ to emphasis that $a,b,c$ are digits base $B$.



                                    Then $[2,9,7]_B mid [7,9,2]_B-[2,9,7]_B$ and we are assuming that $2,9,7 < B$



                                    Writing this out "subtraction-style", we get



                                    $left.begin{array}{c}
                                    & 7 & 9 & 2 \
                                    -& 2 & 9 & 7 \
                                    hline
                                    phantom{4}
                                    end{array}
                                    right.
                                    implies
                                    left.begin{array}{c}
                                    & 6 & (B+8) & (B+2) \
                                    -& 2 & 9 & 7 \
                                    hline
                                    & 4 & (B-1) & (B-5)
                                    end{array}
                                    right.
                                    $



                                    So $[4,B-1,B-5]_B$ is a multiple of $[2,9,7]_B$.



                                    We must therefore have $[4,B-1,B-5]_B = 2[2,9,7]_B = [4,18,14]_B$ which implies $B-1=18$ and $B-5=14$. Hence $B=19$.







                                    share|cite|improve this answer














                                    share|cite|improve this answer



                                    share|cite|improve this answer








                                    edited Jan 2 at 1:56

























                                    answered Jan 1 at 1:35









                                    steven gregorysteven gregory

                                    18.5k32359




                                    18.5k32359






























                                        draft saved

                                        draft discarded




















































                                        Thanks for contributing an answer to Mathematics Stack Exchange!


                                        • Please be sure to answer the question. Provide details and share your research!

                                        But avoid



                                        • Asking for help, clarification, or responding to other answers.

                                        • Making statements based on opinion; back them up with references or personal experience.


                                        Use MathJax to format equations. MathJax reference.


                                        To learn more, see our tips on writing great answers.




                                        draft saved


                                        draft discarded














                                        StackExchange.ready(
                                        function () {
                                        StackExchange.openid.initPostLogin('.new-post-login', 'https%3a%2f%2fmath.stackexchange.com%2fquestions%2f3058019%2ffactors-in-a-different-base-2b2-9b-7-mid-7b2-9b-2%23new-answer', 'question_page');
                                        }
                                        );

                                        Post as a guest















                                        Required, but never shown





















































                                        Required, but never shown














                                        Required, but never shown












                                        Required, but never shown







                                        Required, but never shown

































                                        Required, but never shown














                                        Required, but never shown












                                        Required, but never shown







                                        Required, but never shown







                                        Popular posts from this blog

                                        Biblatex bibliography style without URLs when DOI exists (in Overleaf with Zotero bibliography)

                                        ComboBox Display Member on multiple fields

                                        Is it possible to collect Nectar points via Trainline?